LSAT and Law School Admissions Forum

Get expert LSAT preparation and law school admissions advice from PowerScore Test Preparation.

 Eric Ockert
PowerScore Staff
  • PowerScore Staff
  • Posts: 164
  • Joined: Sep 28, 2011
|
#38509
Hi Pamela!

The question is asking for a pair of people at least one of which must be on the team. In other words, one, or the other, or both of those two variables must be in every possible solution to the game. So answer (D) is saying you must always include either T or Y (or both of them) on the team.

Another way to think of this idea is to think of what CANNOT happen. If you MUST include one of these two, you CANNOT put both of them off of the team. And that is true here. If T is OFF the team, S goes off with him. If Y goes OFF the team, W goes with him (these are based off of the contrapositives of Rule #2 and Rule #3). That leaves only M O P Z left to fill the team, but M can't be selected with O and P (Rule #1). So now there is no way to get at least 4 people on the team. Since it is IMPOSSIBLE to put both T and Y OFF of the team, then you MUST always include one or the other or both ON the team.

Hope that helps!

Get the most out of your LSAT Prep Plus subscription.

Analyze and track your performance with our Testing and Analytics Package.